Jump to content

Photo

Một bài dãy số với lượng giác

- - - - -

  • Please log in to reply
5 replies to this topic

#1
duythuc_dn

duythuc_dn

    Binh nhất

  • Thành viên
  • 24 posts
Có tồn tại hay không 2 dãy số thực ${{a_{i}}},{{b_{i}}} $, $ i\in\mathbb{N} $, thoả mãn các điều kiện sau: $ \dfrac{3\cdot\pi}{2}\leq a_{i}\leq b_{i}$và $\cos(a_{i}x)-\cos(b_{i}x)\geq-\dfrac{1}{i} $ với mọi $\forall i\in\mathbb{N} $ và với mọi 0<x<1?

Edited by duythuc_dn, 03-12-2009 - 16:52.


#2
duythuc_dn

duythuc_dn

    Binh nhất

  • Thành viên
  • 24 posts

Có tồn tại hay không 2 dãy số thực ${{a_{i}}},{{b_{i}}} $, $ i\in\mathbb{N} $, thoả mãn các điều kiện sau: $ \dfrac{3\cdot\pi}{2}\leq a_{i}\leq b_{i}$và $\cos(a_{i}x)-\cos(b_{i}x)\geq-\dfrac{1}{i} $ với mọi $\forall i\in\mathbb{N} $ và với mọi 0<x<1?


Mọi người vào giúp em bài này với. Em cảm ơn.

#3
namdung

namdung

    Thượng úy

  • Hiệp sỹ
  • 1205 posts
Có, cứ chọn $ a_i = b_i $ là xong. Có lẽ tôi hiểu sai đề?

#4
hongthaidhv

hongthaidhv

    GS-TSKHVMF. Lê Hồng Thái

  • Thành viên
  • 442 posts

Có, cứ chọn $ a_i = b_i $ là xong. Có lẽ tôi hiểu sai đề?

Em cũng nghĩ là nếu đề là như thế thì đúng là chỉ cần cho $a_i=b_i, \ \forall i \in N$ thì đk nào cũng thỏa mãn hết( có lẽ đề còn gì thiếu chăng)

Edited by hongthaidhv, 06-12-2009 - 16:37.

M.Lê Hồng Thái
La classe des Matériaux Avancés - Groupe des Écoles des Mines (GEM)
Mél: [email protected]
Y!M: turjnto_le
Facebook: http://www.facebook.com/hongthai.le
Télé: +84(0)936 431 156
+84(0) 979 646 777

#5
vo thanh van

vo thanh van

    Võ Thành Văn

  • Hiệp sỹ
  • 1197 posts
Em nghĩ hay là đề thế này:
Có tồn tại hay không 2 dãy số thực ${{a_{i}}},{{b_{i}}} $, $ i\in\mathbb{N} $, thoả mãn các điều kiện sau: $ \dfrac{3\cdot\pi}{2}\leq a_{i}\leq b_{i}$và $\cos(a_{i}x)+\cos(b_{i}x)\geq-\dfrac{1}{i} $ với mọi $\forall i\in\mathbb{N} $ và với mọi 0<x<1?
Quy ẩn giang hồ

#6
duythuc_dn

duythuc_dn

    Binh nhất

  • Thành viên
  • 24 posts

Em nghĩ hay là đề thế này:
Có tồn tại hay không 2 dãy số thực ${{a_{i}}},{{b_{i}}} $, $ i\in\mathbb{N} $, thoả mãn các điều kiện sau: $ \dfrac{3\cdot\pi}{2}\leq a_{i}\leq b_{i}$và $\cos(a_{i}x)+\cos(b_{i}x)\geq-\dfrac{1}{i} $ với mọi $\forall i\in\mathbb{N} $ và với mọi 0<x<1?

Dạ, đề của anh Văn đúng rồi. Đề này là đúng theo quyển The IMO Compendium ghi.
Còn đề của em lúc đầu là trên MathLinks họ ghi thế.
Mọi người vào làm giúp em bài này với.
Em cảm ơn.




1 user(s) are reading this topic

0 members, 1 guests, 0 anonymous users